Revision as of 16:15, 24 January 2009 by Hyoong (Talk | contribs)

Is there a formal way of saying a<b and c<d implies ac<bd, like a theorem from algebra or something? Just wondering because I used it for my inductive step.

I believe the way you wrote it should be fine for the proof.

Alumni Liaison

Ph.D. on Applied Mathematics in Aug 2007. Involved on applications of image super-resolution to electron microscopy

Francisco Blanco-Silva